review questions Flashcards

1
Q

Which of the following structures would a deep vein thrombus in the leg reach first?
a. aorta
b. L atrium
c. L ventricle
d. pulmonary arteries
e. pulmonary vein

A

d. pulmonary arteries

How well did you know this?
1
Not at all
2
3
4
5
Perfectly
2
Q

Tracer dye is injected into a patient’s skin. Based on the path of tissue fluid flow from the skin, which of the following would the tracer dye reach last and therefore be least likely to show the tracer dye?
a. arteries
b. heart
c. lymphatic vessels
d. lymph nodes
e. veins

A

a. arteries

lymph enters lymphatic capillaries in connective tissue —> enters lymph nodes via afferent lymph vessels and exits via efferent lymph vessels —> lymph drains into venous circulation at R/L venous angles (where subclavian and internal jugular veins meet) via right lymphatic duct and thoracic duct —> enters right side of heart and leaves via pulmonary arteries to lungs

How well did you know this?
1
Not at all
2
3
4
5
Perfectly
3
Q

in a standard radiograph:
a. blood is more radiolucent than fat
b. cartilage is more radiopaque than blood
c. cartilage is more radiopaque than bone
d. enamel is more radiolucent than bone
e. fat is more radiolucent than muscle

A

e. fat is more radiolucent than muscle

radiopacity —> radiolucency
1. foreign heavy metal
2. enamel
3. bone
4. water density (muscle, cartilage, tendon, bone, nerve, connective)
5. fat
6. air

How well did you know this?
1
Not at all
2
3
4
5
Perfectly
4
Q

Faulty neural crest cell migration would result in multiple defects, including defective formation of (2):
a. cardiac septa and cardiac valves
b. the CNS
c. cornea and lens of the eye
d. epithelium of GI and UG systems
e. epithelium of respiratory system
f. ganglia
g. tooth enamel

A

a. cardiac septa and cardiac valves
AND
f. ganglia

neural crest cells form some connective tissues, nervous tissue, muscle tissues, endocrine tissues, melanocytes, odontoblasts (teeth), and cardiac septa & valves

How well did you know this?
1
Not at all
2
3
4
5
Perfectly
5
Q

the cardiovascular system forms from:
a. intermediate mesoderm
b. lateral plate mesoderm, parietal layer
c. lateral plate mesoderm, visceral layer
d. notochord mesoderm
e. paraxial mesoderm

A

c. lateral plate mesoderm, visceral layer

lateral plate mesoderm (cavity):
1. parietal/somatic mesoderm: deep to ectoderm, forms CT of body walls and parietal serosa lining body cavity
2. visceral/splanchnic mesoderm: superficial to endoderm, forms muscles and CT of internal viscera/organs, forms visceral serosa covering viscera/organs

How well did you know this?
1
Not at all
2
3
4
5
Perfectly
6
Q

Obstruction in the LAD would limit blood supply to myocardium of the:
a. anterior 2/3 of the IV septum
b. AV node
c. posterior 1/3 of the IV septum
d. right atrium
e. SA node

A

a. anterior 2/3 of the IV septum

blood supply to ventricular septum:
anterior 2/3 is LCA (left carotid artery)
posterior 1/3 is RCA

How well did you know this?
1
Not at all
2
3
4
5
Perfectly
7
Q

As a result of an endocardial infection, your patient suffers a rupture of the chordae tendineae of the right ventricle. This is likely to result in an alteration of blood flow that will cause a:
a. diastolic murmur that is heard best at the left fifth intercostal space at the sternal border.
b. diastolic murmur that is heard best at the right second intercostal space at the sternal border.
c. systolic murmur that is heard best at the left fifth intercostal space at the midclavicular line.
d. systolic murmur that is heard best at the left second intercostal space at the sternal border.
e. systolic murmur that is heard best at the right fifth intercostal space at the sternal border.

A

e. systolic murmur that is heard best at the right fifth intercostal space at the sternal border.

Chordae tendinae are associated with AV valves, which on on the R is tricuspid valve. AV valves are OPEN during diastole. Ruptured chordae tendinae would impair the ability of AV valves to CLOSE properly (would allow valves to swing open the other way) and so there would be regurgitation… so murmur would be a systolic one, when AV valve should be closed.

Since defect causes regurgitation, “downstream” of the valve is upstream, so you would listen on the right side of the sternal border, rather than the usual auscultation point on the left

How well did you know this?
1
Not at all
2
3
4
5
Perfectly
8
Q

Your patient has suffered a stab wound to the right side of his thorax between the 4th and 5th ribs. Which of the following observations lead you to suspect a tension pneumothorax as opposed to an open pneumothorax?

a. His mediastinum deviates to the right when he exhales.
b. His mediastinum deviates to the right when he inhales.
c. His trachea is deviated to the left and he is cyanotic.
d. His trachea is deviated to the right and he is cyanotic.

A

c. His trachea is deviated to the left and he is cyanotic.

With TENSION PNEUMOTHORAX mediastinal shifts (including the trachea) are to the opposite side of the injury. In this case as air/tension builds in the right pleural space, there is a mediastinal shift to the left, and inhaled air is unable to escape the shifted trachea.

How well did you know this?
1
Not at all
2
3
4
5
Perfectly
9
Q
  1. A baby is born premature at seven months of gestation with a patent ductus arteriosus. A reason that the incidence of patent ductus arteriosus is higher in pre-term babies than it is in full term babies is that the
    a. level of circulating prostaglandin is too low in pre-term babies to allow closure of the ductus.
    b. oxygen level in the blood flowing through the ductus arteriosus in pre-term babies is too high to allow closure of the ductus.
    c. pressure in the pulmonary circulatory system is too low in pre-term babies to allow closure of the ductus.
    d. reversal of the pressure gradient between the right and left sides of the circulatory system does not occur until later in gestation.
    e. underdevelopment of the alveoli of the lung in pre-term babies results in inadequate oxygenation of the blood in the pulmonary circuit.
A

e. underdevelopment of the alveoli of the lung in pre-term babies results in inadequate oxygenation of the blood in the pulmonary circuit.

increased oxygen&bradykinin + decreased prostaglandin causes closure of the ductus arteriosus

How well did you know this?
1
Not at all
2
3
4
5
Perfectly
10
Q

An underweight infant has “tet spells” of cyanosis (turning blue) when he cries and the cyanosis resolves when he is placed in a knee-to-chest position. Based on this presentation, Tetralogy of Fallot (TOF) is suspected and confirmed with further evaluation of a boot-shaped heart on radiographic imaging. TOF results from displacement of the embryonic aorticopulmonary septum, which leads to defects of structures derived from the
a. bulbus cordis.
b. primitive atrium.
c. primitive pulmonary vein.
d. sinus horns.
e. sinus venosus.

A

a. bulbus cordis

How well did you know this?
1
Not at all
2
3
4
5
Perfectly
11
Q

What is a tell-tale radiological sign of Tetralogy of Fallot?

A

boot-shaped heart, due to hypertrophy of the R ventricle

4 defects: pulmonary stenosis, overriding aorta, ventricular septal defect, hypertrophy of R ventricle

How well did you know this?
1
Not at all
2
3
4
5
Perfectly
12
Q

A patient has a solid mass located immediately superior to the root of the right lung. Which structure would most likely be compressed by this mass?

A

arch of the azygos vein - passes superior to the root of the right lung to enter the superior vena cava

How well did you know this?
1
Not at all
2
3
4
5
Perfectly
13
Q

what is the sternal angle a landmark for?

A

sternal angle is landmark for articulation between sternum and costal cartilages of 2nd ribs (anteriorly), T4-T5 (posteriorly), transverse thoracic plane, start/end of the aortic arch, and carina (bifurcation of the trachea)

How well did you know this?
1
Not at all
2
3
4
5
Perfectly
14
Q

at what vertebral level are presynaptic sympathetic neurons found?

A

T1-L2

(presynpatic parasympathetic are S2-S4)

How well did you know this?
1
Not at all
2
3
4
5
Perfectly
15
Q

what is found in dorsal vs ventral root of spinal cord?

A

dorsal root = sensory axons
ventral root = motor axons

merge into mixed spinal nerve, with anterior and posterior ramus (also mixed)

How well did you know this?
1
Not at all
2
3
4
5
Perfectly
16
Q

what nerve loops around the aortic arch

A

left recurrent (read: making a U turn) laryngeal nerve loops around aortic arch to head back up to larynx

How well did you know this?
1
Not at all
2
3
4
5
Perfectly
17
Q

where does blood from the right posterior intercostal veins drain into?

A

azygos vein

How well did you know this?
1
Not at all
2
3
4
5
Perfectly
18
Q

what is the septomarginal trabecula and where is it found (specifically)

A

aka moderator band, found extending from muscular IV septum to anterior papillary muscle in RIGHT ventricle

How well did you know this?
1
Not at all
2
3
4
5
Perfectly
19
Q

what artery courses alongside the great cardiac vein?

A

LAD (left anterior descending) courses along great cardiac vein in anterior IV groove

How well did you know this?
1
Not at all
2
3
4
5
Perfectly
20
Q

what is the fossa ovalis a remnant of?

A

septum primum (the spot where it was not covered by septum secundum)

How well did you know this?
1
Not at all
2
3
4
5
Perfectly
21
Q

what embryologic structure are the AV valves and semilunar valves formed from, respectively?

A

AV valves form from atrioventricular endocardial cushions

semilunar valves form from conotruncal endocardial cushions

How well did you know this?
1
Not at all
2
3
4
5
Perfectly
22
Q

what do the umbilical veins and artery become after birth, respectively

A

umbilical vein —> ligamentum teres
umbilical artery —> medial umbilical ligaments

How well did you know this?
1
Not at all
2
3
4
5
Perfectly
23
Q

where exactly are the following located:
a. SA node
b. AV node
c. AV bundle
d. bundle branches/ Purkinje fibers

A

a. SA node: junction of SVC and R atrium
b. AV node: interatrial septum
c. AV bundle: membranous part of IV septum (IVS)
d. bundle branches/ Purkinje fibers: subendocardium of ventricular walls

How well did you know this?
1
Not at all
2
3
4
5
Perfectly
24
Q

what is carried by the ductus venosus and ductus arteriosus in the fetal heart?

A

ductus venosus carries IVC blood (more oxygenated because mixes with umbilical artery blood)

ductus arteriosus carries SVC blood to arch of aorta after branch points for cranial arteries (not oxygenated)

How well did you know this?
1
Not at all
2
3
4
5
Perfectly
25
Q

CNS develops from ____ while PNS develops from ____

A

neural tube —> CNS (brain/spinal cord)
neural crest cells —> PNS (spinal nerves, cranial nerves, ganglia)

How well did you know this?
1
Not at all
2
3
4
5
Perfectly
26
Q

Explain why it makes sense that ACh signaling (PSNS) slows down HR by causing K+ outflow,

While hyperkalemia causes tachycardia by inducing a more depolarizing state

A

Keep in mind there is usually more K+ inside cell than out, and the inside of the cell is more negative than out (via Na/K ATPase)

ACh causes K+ outflow —> inside of the cell becomes even more negative compared to out —> harder for depolarization to occur

Hyperkalemia (high K+ in blood) lessens the difference between in/out, so K+ within cells is less likely to leave —> inside of the cell becomes more positive —> depolarizing effect

How well did you know this?
1
Not at all
2
3
4
5
Perfectly
27
Q

What part of the ECG does S2 overlay?

A

S2 = semilunar valve closure

corresponds to T wave (ventricular repolarization)

How well did you know this?
1
Not at all
2
3
4
5
Perfectly
28
Q

What part of the ECG does S2 overlay?

A

S2 = semilunar valve closure

overlays T wave (ventricular repolarization)

How well did you know this?
1
Not at all
2
3
4
5
Perfectly
29
Q

Changes in ANS cardiac tone would most profoundly reflect changes in duration of [systole/diastole]

A

diastole

How well did you know this?
1
Not at all
2
3
4
5
Perfectly
30
Q

according to Frank-Starling law, the strength of ventricular contraction is directly proportional to…

A

end-diastolic volume

more volume in means stronger contraction to send it back out

How well did you know this?
1
Not at all
2
3
4
5
Perfectly
31
Q

which of the following is most likely to cause edema?
a. hypotension
b. decreased plasma protein
c. decreased plasma hydrostatic pressure
d. decreased total peripheral resistance

A

b. decreased plasma protein

aka less albumin in blood, therefore less oncotic pressure to pull fluid back into capillaries on venous end

How well did you know this?
1
Not at all
2
3
4
5
Perfectly
32
Q

which of the following most accurately describes CO?
a. stroke volume / TPR
b. heart rate / (EDS - ESV)
c. stroke volume x heart rate

A

CO = SV x HR

How well did you know this?
1
Not at all
2
3
4
5
Perfectly
33
Q

An otherwise healthy patient is experiencing an episode of atrial fibrillation - which of the following is most likely to be measured?
a. abnormally low BP
b. bradycardia
c. reduced ventricular filling
d. impaired CO
e. increased venous return

A

c. reduced ventricular filling

atrial contraction contributes ~20% of ventricular filling

How well did you know this?
1
Not at all
2
3
4
5
Perfectly
34
Q

Damage to the ventral ramus of T1 spinal nerve would affect:
a. somatic fibers and sympathetic fibers
b. parasympathetic motor fibers
c. efferent fibers only
d. afferent fibers only

A

a. somatic fibers and sympathetic fibers

rami of spinal nerves are mixed, but presynaptic sympathetic are present in T1-L2 while parasympathetic are present in S2-4

How well did you know this?
1
Not at all
2
3
4
5
Perfectly
35
Q

Failure of which of the following developmental events would result in cyanosis after birth?
a. ductus arteriosus closure
b. septum primum fusion with septum secundum
c. membraneous interventricular septum fusion with the muscular interventricular septum
d. aorticopulmonary septum formation
e. septum primum fusion with the endocardial cushion tissue

A

d. aorticopulmonary septum formation — failure causes mixture of blood from RV and LV —> cyanosis

How well did you know this?
1
Not at all
2
3
4
5
Perfectly
36
Q

Rupture of the chordae tendinae of the right ventricle would most likely result in:
a. systolic murmur that is heard best at the left second intercostal space at the sternal border
b. systolic murmur that is heard best at the left fifth intercostal space at the midclavicular line
c. systolic murmur that is heard best at the right fifth intercostal space at the sternal border
d. diastolic murmur that is heard best at the right fifth intercostal space at the sternal border
e. diastolic murmur that is heard best at the left second intercostal space at the sternal border

A

c. systolic murmur that is heard best at the right fifth intercostal space at the sternal border

ruptured chordae tendinae —> prolapsed tricuspid valve —> systolic (AV valve should be closed here) murmur at right 5th intercostal space at sternal border (this is regurgitation - downstream of murmur is retrograde)

How well did you know this?
1
Not at all
2
3
4
5
Perfectly
37
Q

Pt is a 47yo F presenting with a Pancoast tumor which is obstructing their SVC, causing edema in their extremities and…
a. increased blood flow to the right atrium
b. pleural effusion
c. jugular venous distention

A

c. JVD - tributary to SVC, therefore would have high pressure if SVC is obstructed

How well did you know this?
1
Not at all
2
3
4
5
Perfectly
38
Q

Pancoast tumors are lung carcinomas that can compress nearby structures, including the recurrent laryngeal nerve, sympathetic stellate ganglion, SVC, brachiocephalic vein, brachial plexus, and phrenic nerve. Which of these would least likely be found in a patient with a Pancoast tumor?
a. mediastinal shift
b. JVD
c. hoarseness of voice
d. sensorimotor deficits in the upper limb
e. hemi-diaphragm paralysis
f. Horner’s syndrome

A

a. mediastinal shift

JVD —> due to SVC compression
voice hoarseness —> due to recurrent laryngeal nerve compression
sensorimotor deficits —> due to compression of brachial plexus
hemi-diaphragm paralysis —> due to phrenic nerve compression
Horner’s syndrome —> due to compression of stellate ganglion of sympathetic chain

How well did you know this?
1
Not at all
2
3
4
5
Perfectly
39
Q

which of these conducts blood with the lowest O2 saturation in the fetus?
a. umbilical vein
b. ductus venosus
c. ductus arteriosus
d. umbilical arteries

A

c. ductus arteriosus

deoxygenated blood from SVC passes through RA—>RV—>pulmonary artery—> ductus arteriosus —> arch of aorta

How well did you know this?
1
Not at all
2
3
4
5
Perfectly
40
Q

Pt is a 64yo F presenting with a diastolic heart murmur best heard at the L 5th intercostal space at the sternal border.

The most likely cause is:
a. incompetent tricuspid valve
b. incompetent mitral valve
c. incompetent pulmonic valve
d. stenotic mitral valve
e. stenotic aortic valve

A

c. incompetent pulmonic valve

incompetent valve causes regurgitation… working backwards from pulmonic valve, “downstream” would be L 5th intercostal space at the sternal border (where tricuspid is normally auscultated)

How well did you know this?
1
Not at all
2
3
4
5
Perfectly
41
Q

Which of the following embryological events accounts for the location of the SA node in the adult heart?
a. replacement of the ostium primum by the ostium secundum
b. neural crest cell migration into the atrioventricular endocardial cushion tissue
c. incorporation of the sinus venosus into the right atrial posterior wall
d. neural crest cell migration into the conotruncal cushion tissue

A

c. incorporation of the sinus venosus into the right atrial posterior wall

heart tube folds via cardiac looping to result in placement of sinus venosus (with nearby SA node) into atria superior to ventricles - on the R side of heart, some of sinus venosus with SA node incorporates into posterior R atrium near the location of the SVC, while on the L side of the heart, the left horn of the sinus venosus incorporates into the heart to form the coronary sinus

How well did you know this?
1
Not at all
2
3
4
5
Perfectly
42
Q

A patient presents with stenosis of the LAD. Their heart is right dominant. Which of the following arteries would there most likely be retrograde blood flow?
a. right coronary artery proximal to the origin of the right marginal artery
b. posterior interventricular (descending) artery proximal to the apex of the heart
c. anterior interventricular (left anterior descending) artery distal to the stenosis
d. right marginal artery

A

c. anterior interventricular (left anterior descending) artery distal to the stenosis

in R coronary artery dominant heart, the posterior descending artery is a branch of the right coronary artery - in this case blood would flow anterograde in the posterior descending artery to reach the LAD where the 2 arteries anastomose - because pressure is lower in LAD distal to stenosis (due to limited blood flow), blood would flow retrograde

How well did you know this?
1
Not at all
2
3
4
5
Perfectly
43
Q

Drug X confers inactive conformation of Na+ channels within type A-delta afferents, allowing only sub-threshold levels of Na+ influx. What kind of drug is this?
a. anti-arrhythmic
b. alpha toxin
c. ACh agonist
d. tetrodotoxin
e. anesthetic

A

e. anesthetic

Type A-deltas are involved in afferent pain relay

How well did you know this?
1
Not at all
2
3
4
5
Perfectly
44
Q

Drug Z acts by prolonging phase 0 of cardiac myocyte AP. Which of the following best describes the mechanism?
a. block Cl- channels
b. activate Ca2+ channels
c. activate K+ channels
d. block Na+ channels
e. up-regulate Na+/K+ ATPase activity

A

d. block Na+ channels

How well did you know this?
1
Not at all
2
3
4
5
Perfectly
45
Q

If left untreated, hyperkalemia can reduce the excitability of muscle cells by which mechanism?
a. lowering the Nernst potential for Na+
b. cell hyperpolarization drives threshold toward 0 mV
c. increased Na+/K+ ATPase activity hyperpolarizes the cells
d. depolarization inactivates voltage-gated Na+ channels
e. raising the electromotive force for K+ efflux from cells

A

d. depolarization inactivates voltage-gated Na+ channels

if severe enough, hyperkalemia can make RMP less negative (more K+ stays in cell because there is so much of it outside the cell - lessens the concentration gradient) —> initially depolarizing effect (allows activation of voltage-gated Na+ channels), but as membrane potential is clamped at this less negative voltage, Na+ channels begin to inactive due to built-in timer (fail safe) mechanism

How well did you know this?
1
Not at all
2
3
4
5
Perfectly
46
Q

A medication working by which of the following mechanisms would most likely reduce a patient’s pulse?
a. inhibition of type 2 cholinergic-muscarinic receptors
b. hyperpolarization of sinoatrial nodal cells
c. inhibition of type alpha 2 adrenoreceptors
d. stimulation of type beta 1 adrenoreceptors
e. inhibition of cardiomyocyte Na+/Ca2+ exchangers

A

b. hyperpolarization of sinoatrial nodal cells —> RMP made more negative —> increased time required to reach threshold —> lower HR

How well did you know this?
1
Not at all
2
3
4
5
Perfectly
47
Q

A drug working by which mechanism would most likely lower a patient’s cardiac sympathetic tone?
a. alpha2 antagonist
b. alpha1 antagonist
c. beta1 antagonist
d. beta2 antagonist
e. cholinomimetic
f. cholinesterase antagonist
g. catecholamine
h. cholinergic-muscarinic agonist

A

c. beta1 antagonist —> this would block SNS activity

How well did you know this?
1
Not at all
2
3
4
5
Perfectly
48
Q

blocking which of the following would reduce contractility of cardiomyocytes?
a. SERCA
b. Na+/K+ ATPase
c. L-type Ca2+ channels
d. Na+/Ca2+ exchanger
e. type 2 cholinergic-muscarinic receptors

A

c. L-type Ca2+ channels

recall that L channels allow for calcium influx, and inotropic state of myocardium is directly proportional to the level of intracellular calcium

How well did you know this?
1
Not at all
2
3
4
5
Perfectly
49
Q

all muscle contractions begin as ______, and only ______ contraction occurs if preload overcomes afterload

A

all muscle contractions begin as ISOMETRIC, and only ISOTONIC contraction occurs if preload overcomes afterload

How well did you know this?
1
Not at all
2
3
4
5
Perfectly
50
Q

Pt is a 73yo F presenting with PE significant for BP of 148/85mmHg, pulse 75/min and regular, RR 15/min. Electrocardiography shows sinus rhythm with QRS duration in lead V1 of 0.14 seconds (ref. 0.06-0.10s) with an R-S-R prime pattern. What is most likely to be seen accompanying these findings?
a. murmur between S2 and S1
b. no discernible P waves
c. complete AV dissociation
d. widened splitting of S2
e. holosystolic murmur best heard at the cardiac apex

A

findings: prolonged QRS, R-S-R pattern (indicating right bundle branch block)

d. widened splitting of S2

How well did you know this?
1
Not at all
2
3
4
5
Perfectly
51
Q

how would MAP, venous pressure, and CO be affected in the first 1-2 seconds following a selective reduction in arterial resistance?

A

reduce arterial resistance —> lower MAP, higher venous pressure, higher CO

reduced resistance allows for increased flow to venous side (higher pressure) and increased venous return to heart

increased venous return + decreased afterload = increased CO

How well did you know this?
1
Not at all
2
3
4
5
Perfectly
52
Q

Which is most likely to result from diastolic heart failure?
a. LV dilation
b. upward shift in cardiac function curve
c. upward shift in diastolic filling curve
d. increased EDV
e. decreased ejection fraction

A

c. upward shift in diastolic filling curve —> IV pressures abnormally elevated
diastolic heart failure = ventricle is not filling efficiently due to decreased compliance (stiff)

a. LV dilation - seen with systolic HF (reduced contractibility)
b. upward shift in cardiac function curve - would indicate higher contractibility
d. increased EDV - seen with systolic HF because of dilation
e. decreased ejection fraction - seen with systolic HF (reduced contractibility)

How well did you know this?
1
Not at all
2
3
4
5
Perfectly
53
Q

systolic vs diastolic heart failure (in basic sense)

A

systolic HF - ventricle is not ejecting as well: eccentric hypertrophy —> reduced contractibility —> reduced ejection fraction

diastolic HF - ventricle is not filling as well: concentric hypertrophy —> reduced compliance —> reduced end diastolic volume

both result in higher ventricular pressure

How well did you know this?
1
Not at all
2
3
4
5
Perfectly
54
Q

A patient with heart failure has reduced SV and increased EDV - how would ejection fraction, ESV, and cardiac SNS tone be affected?

A

reduced SV + increased EDV = systolic heart failure (eccentric hypertrophy, reduced contractibility)

ejection fraction is reduced
end systolic fraction is increased
cardiac SNS tone is increased (attempt to compensate)

How well did you know this?
1
Not at all
2
3
4
5
Perfectly
55
Q

An ECG shows positive QRS in lead I, negative QRS in lead aVF, and positive QRS in lead aVL. What is the best interpretation of these findings?
a. MI
b. LV hypertrophy
c. RAD
d. aortic valve stenosis
e. A fib

A

b. LV hypertrophy - leads are indicated LAD (left axis deviation)

How well did you know this?
1
Not at all
2
3
4
5
Perfectly
56
Q

how would the LV PV loop be shifted from a healthy patient to a patient with poorly managed HTN?

A

LV PV loop = left ventricle pressure volume loop

HTN would shift plot up (higher pressure due to increased afterload) and right (higher end systolic volume)

How well did you know this?
1
Not at all
2
3
4
5
Perfectly
57
Q

what kind of drug is Enalapril?

A

ACE inhibitor —> induces vasodilation by inhibiting RAAS system

How well did you know this?
1
Not at all
2
3
4
5
Perfectly
58
Q

A patient is prescribed an antiarrhythmic drug to manage their paroxysmal atrial fibrillation. On follow-up, their ECG shows regular rhythm with prolonged QT intervals. What is the most likely mechanism of this electrophysiology anomaly?
a. decreased expression of voltage-gated Na+ channels
b. decreased duration of If (funny current)
c. impeding the activation of potassium current
d. hypercalcemia
e. formation of a reentrant loop

A

c. impeding the activation of potassium current - by impeding repolarizing K+ currents, duration of AP is prolonged

How well did you know this?
1
Not at all
2
3
4
5
Perfectly
59
Q

Pt is a 75yo M presenting with chest and jaw pain for 2 days duration. Nitroglycerine provides some relief. PE shows BP is 140/85mmHg and pulse is 88/min and regular. Which of the following ECG findings would corroborate his signs and symptoms?
a. fibrillatory waves in lead I
b. prolonged PR interval in leads II and III
c. inverted P waves in several precordial leads
d. QRS negative in lead I, positive in lead aVF
e. ST elevation in leads V3 and V4

A

e. ST elevation in leads V3 and V4

signs/symptoms indicate myocardial infarction (MI) - ST elevation is key finding

How well did you know this?
1
Not at all
2
3
4
5
Perfectly
60
Q

a patient presents with severe hyperkalemia - which of these ECG findings is likely?
a. V fib
b. prominent Q waves
c. PR shortening
d. tented T waves
e. inverted P waves

A

d. tented T waves

remember that imbalances of K+ will cause issues with repolarization, while imbalances of Ca2+ will cause issues with depolarization (QRS)

How well did you know this?
1
Not at all
2
3
4
5
Perfectly
61
Q

how will 30 minutes of exercise cause a shift in the LV PV loop?

A

LV PV loop = left ventricle pressure volume loop

exercise will cause shift up (higher BP —> higher afterload —> higher LV pressure) and left (higher contractibility —> higher SV —> lower ESV)

How well did you know this?
1
Not at all
2
3
4
5
Perfectly
62
Q

when do AV valves close

A

S1 sound, at the time of isovolumetric contraction (~middle of QRS wave)

How well did you know this?
1
Not at all
2
3
4
5
Perfectly
63
Q

what ECG would you suspect to accompany a patient with junctional escape and retrograde atrial conduction, in leads II and aVF?

A

inverted P waves (should be positive in leads II and aVF) —> retrograde conduction is causing depolarization to move away from the leads

How well did you know this?
1
Not at all
2
3
4
5
Perfectly
64
Q

3 ways to increase preload on the heart

A
  1. add volume
  2. slow heart rate (more time for filling)
  3. constrict veins (more venous return)*

*this is why the response to blood loss is venous constriction (veins hold a large blood volume)

How well did you know this?
1
Not at all
2
3
4
5
Perfectly
65
Q

what is the general mechanism of nitrates on the CV system?

A

nitrates cause blood to pool in veins —> less venous return —> decrease preload —> lower work for heart

66
Q

how do the following parameters change in response to exercise?
a. SBP
b. DBP
c. TPR
d. PP

A

a. SBP (systolic BP): increases due to higher CO (primary determinant of SBP is CO)
b. DBP (diastolic B): decreases slightly due to vasodilation in skeletal muscle (primary determinant of DBP is TPR)
c. TPR (total peripheral resistance): decreases due to vasodilation in skeletal muscle
d. PP (pulse pressure): increases due to larger difference between SBP and DBP

67
Q

when would HR increase CO vs decrease CO?

A

CO = SV x HR

normally an increase in heart rate raises cardiac output,
but in pathological state, increased heart rate can lower cardiac output if stroke volume can’t keep up

(relationship between CO and HR looks like an inverted U)

68
Q

Peak left ventricular contraction will occur during what phase of the ECG?

A

Plateau phase of myocardial AP (RIGHT after QRS complex)

69
Q

What kind of drugs are lisinopril and amlodipine, respectively?

A

Lisinopril: ACE inhibitor

Amlodipine: Ca2+ channel blocker

70
Q

What kind of drugs are enalapril and amlodipine?

A

Enalapril: ACE inhibitor

Amlodipine: Ca2+ channel blocker

71
Q

What is the effect of nitroglycerine?

A

Metabolized into NO —> vasodilation

72
Q

what are the classic causes of restrictive vs dilated cardiomyopathy?

A

restricted: Puppy LEASH: Post-radiation fibrosis, Loeffler’s, Endocardial fibroelastosis, Amyloidosis, Sarcoidosis, Hemochromatosis

dilated: ABCCCD: Alcohol, Beriberi (thiamine deficiency), Cocaine, Chagas, Coxsackievirus, Doxorubicin

73
Q

A 56yo with HTN is prescribed amlodipine. Three months later they return for evaluation. Which of the following new symptoms could be attributed to this new drug regime?
a. persistent cough
b. Lupus-like rash
c. postural hypotension
d. palpitations
e. diarrhea

A

c. postural hypotension - amlodipine is a calcium channel blocker, which cause peripheral arteriolar vasodilation - reduces the patient’s ability to compensate from sitting to standing with vasoconstriction and this will cause BP to drop with positional change —> postural hypotension

a. persistent cough - due to ACE inhibitors
b. Lupus-like rash - due to procainamide

74
Q

A 66yo with a PMH of COPD is seen for a routine check-up. Their BP measurement is 157/90 on three separate measurements. In addition to lifestyle recommendations, they are prescribed amlodipine. Reduced blood pressure can be achieved with this drug because it inhibits:
a. Angiotensin 2 receptors
b. L-type calcium channels
c. alpha 1 adrenergic receptors
d. renal sodium reabsorption

A

b. L-type calcium channels

Amlodipine is an L-type calcium channel blocker in vascular smooth muscle cells

75
Q

A 29yo male firefighter presents to his cardiologist with dyspnea on exertion. He reports he is having trouble keeping up with the squad and can no longer carry his 40-lb pack. He reports nocturnal cough, a 10-lb weight loss and fatigue. He has not travelled outside the US. PE reveals elevated JVP and decreased breath sounds at the bases bilaterally. Leftward displacement of the PMI and an S3 are noted. He has a holosystolic murmur at the apex, radiating to the axilla. Echo reveals left ventricular systolic dysfunction and four-chamber dilation. Which of the following is most likely the cause of his heart disease?
a. undiagnosed HTN
b. stress cardiomyopathy
c. amyloid deposition
d. alcohol abuse
e. prior strep infection

A

d. alcohol abuse

nocturnal cough + DOE (dyspnea on exertion) + bi-ventricular dilation = dilated cardiomyopathy
most common causes of dilated cardiomyopathy are viral/parasitic but these are not choices, therefore alcohol is the best answer

(a.) undiagnosed HTN and (c.) amyloid deposition cause restrictive cardiomyopathy
(e.) prior strep infection causes mitral stenosis

76
Q

A 29yo male firefighter presents to his cardiologist with dyspnea on exertion. He reports he is having trouble keeping up with the squad and can no longer carry his 40-lb pack. He reports nocturnal cough, a 10-lb weight loss and fatigue. He has not travelled outside the US. PE reveals elevated JVP and decreased breath sounds at the bases bilaterally. Leftward displacement of the PMI and an S3 are noted. He has a holosystolic murmur at the apex, radiating to the axilla. Echo reveals left ventricular systolic dysfunction and four-chamber dilation. For this patient, which of the following would be true regarding their expected pressure-volume curve?
a. the ESPVR slope is decreased and ESV is decreased
b. the EDPVR slope is increased and the ESV is decreased
c. the ESPVR slope is decreased and the ESV is increased
d. the ESPVR slope is increased and the ESV is increased
e. the EDPVR slope is increased and the ESV is increased

A

c. the ESPVR slope is decreased and the ESV is increased

patient is presenting with dilated cardiomyopathy, therefore their LV is less able to eject blood effectively —> decreased ESPVR slope and increased ESV

77
Q

A 46yo with no significant PMH presents to the ED with progressive dyspnea on exertion. She reports recent orthopnea as well as lower extremity swelling. On PE, the pulse is 110/min and BP is 90/60mmHg. JVP is elevated. Lungs are clear. S1 and S2 are audible, accompanied by an S4. Echo reveals a normal EF; however, there is biventricular wall thickening with normal cavity size and bi-atrial enlargement. The myocardium appears “speckled.” A myocardial biopsy is taken and treated with Congo Red stain, which confirms diagnostic suspicion. Which of the following additional findings would be consistent with the suspected diagnosis?
a. an “opening snap”
b. ascites and a tender liver
c. lateral movement of the PMI

A

b. ascites and a tender liver

pt is presenting with restrictive cardiomyopathy: produces biventricular thickening and atrial enlargement
Congo Red stain is used to detect amyloid deposition

a. an “opening snap” = mitral stenosis
c. lateral movement of the PMI = large volume in the ventricle

78
Q

A 21yo M is evaluated at his yearly physical. He has no medical problems and takes no medications. On PE, BP is 128/73mmHg, pulse rate is 56/min. There is a grade 2/6 early systolic murmur along the left sternal border that worsens with Valsalva. An S4 is present. On further questioning, he reports experiencing dizziness while exercising. An echo shows marked septal hypertrophy with an EF of 65%, marked atrial enlargement, and reduced early diastolic filling. Which of the following pathologic findings would be consistent with this likely disorder?
a. iron deposition in the myocardium
b. immunoglobin light chain deposition in the myocardium
c. lymphocyte infiltration of the myocardium
d. Aschoff bodies in the cardiomyocytes
e. disarray of the cardiomyocytes with fibrosis

A

e. disarray of the cardiomyocytes with fibrosis = hypertrophic obstructive cardiomyopathy

a. iron deposition in the myocardium = hemosiderosis
b. immunoglobin light chain deposition in the myocardium = amyloidosis (—> restrictive cardiomyopathy)
c. lymphocyte infiltration of the myocardium = viral dilated cardiomyopathy
d. Aschoff bodies in the cardiomyocytes = rheumatic heart disease

79
Q

Which of these worsens systolic heart function?
a. hypertrophic cardiomyopathy
b. long-standing HTN
c. cardiac amyloidosis
d. aortic stenosis
e. ischemic injury to the heart

A

d. aortic stenosis —> systolic dysfunction

the following worse diastolic function:
a. hypertrophic cardiomyopathy
b. long-standing HTN
c. cardiac amyloidosis (—> restrictive cardiomyopathy)
e. ischemic injury to the heart

80
Q

If you’re looking at a PV loop of the left ventricle, how would you calculate ejection fraction?

A

determine EDV and ESV, then subtract ESV from EDV - then divide by EDV

basically finding what fraction of the total available blood was actually ejected out

81
Q

A 6yo boy has a history of corneal opacities and relapsing polyneuropathy. His lipid profile shows normal cholesterol, low HDL, and normal triglycerides. Which of the following is LEAST likely to be deficient in this patient?
a. ABCA1
b. ApoA1
c. LCAT
d. LDL receptor

A

d. LDL receptor

hypoalphalipoproteinemia: low plasma HDL, caused by deficiency in either ABCA1 (cholesterol transporter in peripheral tissue —> Tangier disease), LCAT (esterifies cholesterol), or ApoA1 (associated with HDL)

82
Q

cholestyramine is a bile acid sequestrant which acts to lower cholesterol by inhibiting:
a. HMG Co-A reductase
b. hepatic cholesterol synthesis
c. release of bile salts from the liver
d. enterohepatic circulation (reabsorption) of bile salts
e. abortion of dietary cholesterol by NPC1L1

A

d. enterohepatic circulation (reabsorption) of bile salts

bile acid sequestrants SEQUESTER bile salts so they cannot be reabsorbed - this causes increased synthesis of bile salts, which uses up cholesterol (precursor), thereby lowering levels of free plasma cholesterol

83
Q

A 16yo boy is being evaluated. The femoral pulse is diminished in amplitude and delayed compared to the brachial pulse. There is an early systolic ejection click at the 2nd ICS to the right of the sternum. What is the likely diagnosis?

A

aortic coarctation (delay in femoral pulse) with bicuspid aortic valve (ejection click)

50% of patients with coarctation have a bicuspid valve

84
Q

A 16yo boy is being evaluated. The femoral pulse is diminished in amplitude and delayed compared to the brachial pulse. There is an early systolic ejection click at the 2nd ICS to the right of the sternum. Which of the following additional findings would further support your presumptive diagnosis?
a. pulsatile mass in upper abdomen
b. continuous murmur over the scapulae
c. holosystolic murmur at the apex
d. absent left carotid pulse

A

b. continuous murmur over the scapulae

pt has aortic coarctation with bicuspid aortic valve - continuous murmur over scapulae reflects development of intercostal collaterals

85
Q

what is the mechanism of action of milrinone?
a. blocks L-type calcium channels
b. inhibits phosphodiesterase 3
c. blocks Na+/K+ ATPase pump
d. blocks alpha adrenergic receptors

A

b. inhibits phosphodiesterase 3 (PDE3) —> blocks degradation of cAMP —> increased inotropy in the heart, peripheral vasodilator

86
Q

what does laterally displaced PMI indicate

A

volume overload —> eccentric hypertrophy

systolic HF, ischemic heart disease, valve disease, dilated cardiomyopathy

seen with S3 heart sound

87
Q

what does RV or LV heave indicate

A

pressure overload —> concentric hypertrophy

aortic stenosis, HTN, diastolic HF

seen with S4 heart sound

88
Q

what does a loud P2 heart sound indicate

A

increased pulmonary artery pressure

high LA or LV pressure, primary lung disease

89
Q

what does wide-fixed splitting of S2 heart sound indicate?

A

atrial septal defect

90
Q

what does wide physiological splitting of S2 heart sound indicate

A

delayed P2

due to RBBB or pulmonic stenosis

91
Q

what does paradoxical splitting of S2 heart sound indicate?

A

delayed A2

due to LBBB or aortic stenosis

92
Q

what does early systolic click indicate

A

bicuspid aortic valve

seen with aortic stenosis in a young person

93
Q

what does mid-systolic click indicate

A

mitral valve prolapse

moves earlier in systole with decreased volume in the heart

94
Q

what does an opening snap indicate in cardiac auscultation

A

opening of mitral valve in mitral valve stenosis

95
Q

what does pericardial knock indicate

A

cessation of ventricular filling in constrictive pericarditis

96
Q

what does scratchy friction rub in cardiac auscultation indicate

A

acute pericarditis

97
Q

what does pulsus parvus et tardus indicate

A

delayed + weak carotid pulse

due to aortic stenosis

98
Q

what does wide pulse pressure indicate

A

high SBP with low DBP

aortic regurgitation, AV fistula

99
Q

what does pulsus paradoxus indicate

A

drop in SBP with inspiration

due to early pericardial tamponade

100
Q

when do heart murmurs INCREASE with Valsalva maneuver?

A

murmurs that are worse when heart is small:
1. mitral valve prolapse
2. hypertrophic obstructive cardiomyopathy

101
Q

which murmurs (2) increase with inspiration

A
  1. tricuspid regurgitation
  2. pulmonic stenosis
102
Q

what does continuous “machine-like” murmur indicate

A

constant flow in diastole and systole

due to patent ductus arteriosus

103
Q

what kind of cardiac issue does rales or crackles in the lungs indicate

A

high pulmonary capillary pressures

due to left heart failure

104
Q

what does negative lead I and positive lead II indicate on ECG?

A

right axis deviation (RAD)

105
Q

what does wide QRS vs prolonged QT interval indicate

A

wide QRS = conduction problem (BBB or Na+ channel problem)

prolonged QT = slow K+ current or prolonged Ca2+ current (medications, low serum calcium, K+ channel blockers)

106
Q

match:
early afterdepolarizations (EAD) vs delayed afterdepolarizations (DAD)
with
hypercalcemia vs hypokalemia

A

hypokalemia —> early afterdepolarizations (EAD): occur during phase 2 or 3
*can be caused by prolonged QT interval

hypercalcemia —> delayed afterdepolarizations (DADs): occur during phase 4
*digoxin can induce DADs, which makes sense because digoxin causes increased intracellular Ca2+!

107
Q

why does it make sense that ischemic tissue depolarizes more?

A

after myocardial infarction, there is failure of Na+/K+ ATPase because there is no ATP! Without this function, Na+ builds up in the cell and causes depolarization and cell swelling

there will also be a buildup of Ca2+ in the cytosol because SERCA (sarcoplasmic reticulum calcium transport ATPase) also relies on ATP

108
Q

what are the preferred anti hypertensive drugs in pregnancy?

A

Hypertensive Moms Love NIFEDIPINE

Hydrazine
Methyl-dopa alpha
Labetalol
Nifedipine

109
Q

Which of the following is most likely to result from diastolic HF?
a. LV dilation
b. upward shift in the cardiac function curve
c. upward shift in the diastolic filling curve
d. increased EDV
e. decreased EF

A

diastolic HF = stiff LV, hard to fill (less compliant)

c. upward shift in the diastolic filling curve = higher pressure during diastole

110
Q

how will the following parameters be affected by heart failure with reduced SV and increased EDV?
a. ejection fraction
b. end systolic pressure
c. cardiac SNS tone

A

HF with reduced SV and EDV = systolic HF

a. EF is LOW
b. end systolic pressure is LOW - heart cannot generate the pre load necessary to situation adequate SV
c. cardiac SNS tone is HIGH

111
Q

how would poorly managed HTN affected the LVPV plot?

A

LVPV (left ventricle pressure-volume) plot would be shifted UPward and to the RIGHT

pressure would be increased (upward shift) working against greater afterload and end systolic volume would be increased also because of this (right shift = less emptying)

112
Q

how would 20 minutes of exercise alter the LV pressure-volume plot?
a. upward shift in diastolic filling curve
b. ESPVR shifted upward and leftward
c. ESV increased
d. EDV decreased
e. decreased slope of ESPVR

A

b. ESPVR shifted upward and leftward

ESPVR curve shows inotropic state of LV - with exercise, inotropy is increased with SNS tone, so there would be an increased slope, and leftward shift = more emptying (lower ESV) due to greater contractility

113
Q

when does PEAK FORCE OF CONTRACTION of left ventricular myocytes occur in ECG reading?

A

PEAK contraction occurs during PLATEAU phase of myocardial AP

114
Q

A 52yo M recently emigrated from Southeast Asia presents with worsening dyspnea on exertion and night-time cough. His PE is significant for JVD and bibasilar lung crackles. PMI is non-displaced. P2 is prominent. There is an early diastolic “opening snap” followed by a decrescendo murmur at the apex, best heart in the left lateral ducubitus position using the bell. Pre-systolic accentuation is present. Which of the following would indicate this valve pathology has worsened over time?
a. widened pulse pressure
b. increased murmur intensity
c. increased S1 intensity
d. presence of an S4
e. shorter S2-opening snap interval

A

e. shorter S2-opening snap interval

patient is presenting with mitral stenosis (probably Rheumatic fever) —> increased LA pressure leads to earlier opening of the valve over time (pressure in LA becomes higher than LV pressure sooner) —> shorter S2-opening snap interval

115
Q

An early systolic ejection click with a late-peaking systolic ejection murmur loudest at the R 2nd ICS would most likely also present with:
a. water hammer pulses in the carotid
b. S3
c. paradoxical splitting of the 2nd heart sound
d. systolic BP of 150 and diastolic BP of 50

A

c. paradoxical splitting of the 2nd heart sound

early systolic ejection click + late-peaking systolic ejection murmur at R 2nd ICS = aortic stenosis

this would cause aortic valve to close later —> A2 is heard after P2 (paradoxical splitting)

116
Q

what valve defects are associated with the following findings:
a. opening snap
b. pulsus parvus et tardus
c. mid-systolic click
d. wide pulse pressure
e. paradoxical splitting of S2

A

a. opening snap = mitral stenosis
b. pulsus parvus et tardus = aortic stenosis
c. mid-systolic click = mitral valve prolapse
d. wide pulse pressure = aortic regurgitation
e. paradoxical splitting of S2 = aortic stenosis

117
Q

a holosystolic blowing murmur heard loudest at the apex and radiating to the axilla would most likely also present with:
a. weak and delayed carotid pulse
b. early systolic ejection click
c. increased murmur intensity during inspiration
d. S3
e. wide pulse pressure

A

mitral regurgitation would most likely also present with S3 (it’s throwing all that extra volume back into the left ventricle)

118
Q

which murmur will you hear?
35yo with recent history of viral myocarditis and dilated cardiomyopathy presenting with worsening dyspnea on exertion

A

mitral regurgitation (due to dilation)

119
Q

which murmur will you hear?
45yo with PMH of Marfan’s presents with exertional dyspnea and fatigue

A

aortic regurgitation (likely due to ascending aortic aneurysm which dilates aortic root)

120
Q

which murmur will you hear?
54yo reports 2 episodes of exertional syncope and recent onset of chest pain. He has PMH of HTN

A

aortic stenosis (due to calcification)

121
Q

which murmur will you hear?
50yo with PMH of rheumatic heart disease presents with dyspnea on exertion and fatigue. The symptoms have worsened over the past few weeks.

A

mitral stenosis (due to Rheumatic heart disease)

122
Q

which murmur will you hear?
30yo F presents with episodes of mild SOB and palpitations; however, it does not impair daily living. She is otherwise healthy

A

mitral prolapse (least symptomatic)

123
Q

which murmur will you hear?
mid-diastolic rumble following opening snap with pre-systolic accentuation

A

mitral stenosis

124
Q

which murmur will you hear?
mid-systolic click followed by a murmur, heard best at apex

A

mitral prolapse

125
Q

which murmur will you hear?
holosystolic murmur, heard best at L 5th ICS in the MCL that radiates to the axilla

A

mitral regurgitation

126
Q

which murmur will you hear?
high pitched early diastolic decrescendo murmur that is loudest at lower left sternal border

A

aortic regurgitation

127
Q

which murmur will you hear?
mid-systolic crescendo-decrescendo murmur heard best at R 2nd ICS that radiates to the neck

A

aorta stenosis

128
Q

which murmur will you hear?
additional signs include pulsus parvus et tardus and S4

A

aortic stenosis

129
Q

which murmur will you hear?
additional signs include lateral shift of PMI and S3

A

mitral regurgitation

130
Q

which murmur will you hear?
additional signs include difficulty swallowing and prominent P2 component of S2

A

mitral stenosis

131
Q

which murmur will you hear?
additional signs include the patient being otherwise healthy

A

mitral prolapse

132
Q

which murmur will you hear?
additional signs include wide pulse pressure, capillary pulsations in the nail beds, “water hammer” pulse over carotid artery

A

aortic regurgitation

133
Q

which murmur will you hear?
calcification of mitral valve in “fish mouth” deformity

A

mitral stenosis

134
Q

which murmur will you hear?
concentric hypertrophy of LV, cardiomyocytes have sarcomeres assembled in parallel

A

aortic stenosis

135
Q

which murmur will you hear?
there is myxomatous degeneration of the mitral valve

A

mitral prolapse

136
Q

which murmur will you hear?
aortic histopathology shows collagen fragmentation and loss of fibrillin expression

A

aortic regurgitation (due to Marfan’s)

137
Q

which murmur will you hear?
myocardial biopsy shows diffuse interstitial infiltration with lymphocytes

A

mitral regurgitation (due to DILATED myocarditis)

138
Q

which murmur will you hear?
echocardiogram shows displacement of mitral leaflet into the LA during midsystole

A

mitral prolapse

139
Q

which murmur will you hear?
echocardiogram shows LA dilation and mitral valve leaflets are thickened

A

mitral stenosis

140
Q

which murmur will you hear?
echocardiogram shows an ascending aortic aneurysm which dilates the aortic root

A

aortic regurgitation

141
Q

why would a beta blocker be useful in a patient with mitral valve stenosis?

A

beta blocker would slow HR, allowing for more diastolic filling time

142
Q

what causes exercise-induced syncope with aortic stenosis?

A

exercise induces increase in arterial vasodilation, which requires more CO - but this cannot be provided by heart with aortic stenosis, so syncope can occur

143
Q

what does early systolic ejection click heard with aortic stenosis indicate?

A

bicuspid aortic valve - usual cause of aortic stenosis when patient is young

144
Q

Pt is a previously healthy 55yo presenting with progressive exertional dyspnea and exercise intolerance and recent-onset orthopnea. PE is remarkable for laterally displaced PMI, soft S1, and early diastolic decrescendo murmur beginning at S2. The murmur is heard best along the L sternal border with the patient seated and leaning forward. What murmur is this?

A

aortic regurgitation - this will cause eccentric hypertrophy (sarcomeres in series)

145
Q

why might aortic regurgitation result in reduced coronary perfusion pressure?

A

aortic regurgitation causes very low diastolic BP, and coronary perfusion is reliant on DBP (pressure in aorta during diastole)

146
Q

which papillary muscle has dual blood supply?

A

posteromedial papillary muscle (LAD and RCA)

147
Q

pt presents with palpitations and feeling of “heart jumping out of chest;” PE is significant for mid-systolic click followed by high pitched systolic murmur that is loudest at the apex. The primary pathogenesis of this valvular disease is most commonly associated with:
a. dilation of the valve annulus
b. calcium deposits in the valve annulus
c. myxomatous degeneration of the valve leaflets

A

c. myxomatous degeneration of the valve leaflets

collagen fragments and is replaced by proteoglycans

(valve disease is mitral prolapse)

148
Q

what murmur is this?
65yo with PMH of pulmonary fibrosis, complaining of increased fatigue. PE reveals JVD, bilateral lower extremity edema, holosystolic murmur that is loudest at L sternal border and increases with inspiration.

A

tricuspid regurgitation: most often caused by annulus dilation (due to RV dilation), unless endocarditis is suspected

note JVD/peripheral edema and increasing with inspiration points to a R heart problem
also holosystolic = regurgitation (stenosis murmurs fade out)

149
Q

how could you distinguish between a ventral septal defect murmur and tricuspid regurgitation? (how could you manipulate the murmurs?)

A

both are holosystolic murmurs, but only tricuspid regurgitation will increase with inspiration

increases with inspiration? Must be tricuspid regurgitation!

150
Q

where is the most common trigger location for a-fib (and where ablation therapy is performed)?

A

pulmonary veins

151
Q

what kind of murmur does atrial vs ventricular septal defect cause?

A

atrial septal defect = fixed splitting of S2

ventricular septal defect = holosystolic murmur at left sternal border (will NOT increase with inspiration as tricuspid regurgitation would)

152
Q

what kind of heart block can be caused by right coronary artery occlusion?

A

1st degree heart block and 2nd degree Mobitz Type 1 heart block: caused by SA/AV node block

2nd degree Mobitz Type II heart blocks are caused by blocks in bundle of His, which is supplied by LAD!

153
Q

which classes of antiarrhythmic drugs are for rate control, which are for rhythm control?

A

rhythm control: Class 1 (block Na+) and Class 3 (block K+)

rate control: Class 2 (beta blockers) and Class 4 (block Ca2+)

154
Q

match:
Streptococcus infection or Staphylococcus infection
with
tricuspid valve defect and mitral valve defect

A

Strep A infection —> rheumatic fever —> mitral valve defects

Staphylococcus infection (IV drug user) —> infective endocarditis —> tricuspid valve defects (first valve the infection hits when it reaches heart)

155
Q

Which of these drugs has the potential to precipitate polymorphic ventricular tachycardia?
a. metoprolol
b. lidocaine
c. diltiazem
d. sotolol

A

d. sotolol: class 3 antiarrhythmic, blocks K+, thereby prolonging QT interval —> risk of polymorphic ventricular tachycardia / torsade des pointes

156
Q

which anti-arrhythmic drug is most commonly associated with development of Lupus-like syndrome?

A

Procainamide (Class 1A - block Na+)

157
Q

what are cannon A waves associated with?

A

AV node dissociation - due to atria and ventricle contracting at the same time

158
Q

Pt. presents with heart “jumping in their chest” due to abrupt onset of palpitations. QRS is narrow and tachycardic with regular rate of 160/min. No P waves are evident. A vagal maneuver terminates the arrhythmia. What is most likely the diagnosis?
a. AVRT
b. atrial flutter with 2:1 conduction block
c. ventricular tachycardia
d. AVNRT

A

d. AVNRT - remember that vagal maneuvers will only be effective on arrhythmias involving AV node

AVNRT is more common than AVRT

159
Q

polymorphic ventricular tachycardia is associated with [EADs/DADs]?

A

polymorphic ventricular tachycardia is associated with early afterdepolarizations

160
Q

tendinous xanthomas are most often associated with deficiency in:
a. LDL receptor
b. LPL
c. ApoB100
d. ApoE

A

a. LDL receptor deficiency

tendinous xanthomas seen with familial hypercholesterolemia (can also be caused by ApoB100 mutations, but far less common)

161
Q

which of the following would increase the murmur of hypertrophic cardiomyopathy?
a. hand grip
b. inspiration —> expiration
c. standing to squatting
d. positive inotrope

A

d. positive inotrope —> would DECREASE volume in the heart

hypertrophic cardiomyopathy murmur (obstructive murmur due to septal hypertrophy) gets louder with LESS volume in heart

162
Q

A 16yo M presents with right arm BP of 180/95mmHg and delayed/diminished femoral pulse as compared to brachial pulse. Which of the following additional findings would support the diagnosis?
a. pulsatile mass in upper abdomen
b. continuous murmur over the scapulae
c. holosystolic murmur at the apex
d. absent L carotid pulse
e. capillary nail bed perfusions

A

patient has aortic contraction - continuous murmur over the scapulae (b) would indicate development of intercostal collaterals

a. pulsatile mass in upper abdomen = AAA
c. holosystolic murmur at the apex = ventricular septal defect or mitral stenosis
d. absent L carotid pulse = aortic dissection
e. capillary nail bed perfusions = aortic regurgitation